Change of Variables(nonlinear function to a linear one)

Click For Summary
The discussion focuses on converting a nonlinear model, specifically f(x) = ax/(b+x), into a linear form suitable for regression analysis. The user attempts to apply a change of variables, initially trying to take the reciprocal of both sides and then experimenting with logarithmic transformations, but encounters difficulties due to the variable x being in the denominator. They also propose rewriting the equation in a matrix format to facilitate the linearization process. The user seeks assistance in successfully transforming the function to achieve a linear regression model. Effective change of variables is crucial for applying linear regression to nonlinear data.
HclGuy
Messages
13
Reaction score
0

Homework Statement


I'm given a nonlinear model which I am trying to perform a linear regression on. I need to use change of variables in order to convert the nonlinear model into a linear one.

Homework Equations


I am given f(x) = \frac{ax}{b+x} where a and b are just parameters.
I need to use change of varibles to change this function into a form of F(x) = AX + B

The Attempt at a Solution


I have tried to take the reciprocal of both sides to get
\frac{1}{y} = \frac{b+x}{ax} = \frac{b}{ax} + \frac{1}{a} , only problem with that is that the x is in the denominator. I have also tried using log transformation to both sides without success. Any help is appreciated thanks!
 
Physics news on Phys.org
see below
 
Last edited:
I would rewrite it as b*f(x) - ax = x*f(x)

or in matrix form

\left(\begin{array}{cc}x_{1}&f(x_{1})\\x_{2}&f(x{2})\\.&.\\.&.\\.&.\\x_{n}&f(x_{n})\end{array}\right) \left(\begin{array}{cc}-a\\b\end{array}\right) = \left(\begin{array}{cc}x{1}*f(x_{1})\\x{2}*f(x{2})\\.\\.\\.\\x_{n}*f(x_{n})\end{array}\right)
 
Question: A clock's minute hand has length 4 and its hour hand has length 3. What is the distance between the tips at the moment when it is increasing most rapidly?(Putnam Exam Question) Answer: Making assumption that both the hands moves at constant angular velocities, the answer is ## \sqrt{7} .## But don't you think this assumption is somewhat doubtful and wrong?

Similar threads

Replies
6
Views
1K
  • · Replies 13 ·
Replies
13
Views
2K
Replies
8
Views
2K
Replies
1
Views
2K
  • · Replies 2 ·
Replies
2
Views
933
  • · Replies 5 ·
Replies
5
Views
1K
  • · Replies 8 ·
Replies
8
Views
2K
  • · Replies 3 ·
Replies
3
Views
2K
  • · Replies 22 ·
Replies
22
Views
4K
  • · Replies 8 ·
Replies
8
Views
1K